What is the Absolute Value of an Exponential with Complex Exponent?

Click For Summary
The discussion focuses on calculating the absolute value of the expression |e^{a^{2} + \frac{it}{m\hbar}}|^{2}. It is established that the complex exponent can be separated into its real and imaginary parts, leading to the conclusion that the absolute value of the exponential function with an imaginary exponent is 1, according to Euler's formula. The correct calculation ultimately simplifies to |e^{(real part)}|^2 = e^{2a^{2}}. Participants clarify misconceptions about the properties of absolute values in relation to complex exponentials. The final agreement confirms the solution as e^{2a^{2}}.
arierreF
Messages
78
Reaction score
0
I want to calculate |e^{a^{2} + \frac{it}{m\hbar}}|^{2}


i is imaginary unit.


my trie:

a^{2} + \frac{it}{2m\hbar} is a complex number so its module is:


\sqrt{a^{4} + \frac{t^{2}}{m^{2}\hbar^{2}}}

= \sqrt{a^{4}(1 + \frac{t^{2}}{m^{2}\hbar^{2}a^{4}})}

a^2\sqrt{1 + \frac{t^{2}}{m^{2}\hbar^{2}a^{4}}}


So the solution is:


(e^{a^2\sqrt{1 + \frac{t^{2}}{m^{2}\hbar^{2}a^{4}}}})^{2}

=
e^{2a^2\sqrt{1 + \frac{t^{2}}{m^{2}\hbar^{2}a^{4}}}}




My friend said that is is definitely wrong. And i actually think that it is wrong.

can somebody tell me where?
 
Physics news on Phys.org
Is there a theorem that says that ##|e^z|=e^{|z|}## for all complex numbers z? Since I'm asking, you can guess that the answer is no. I suggest that you verify this by finding a counterexample.

Do you know anything about the exponential function that would allow you to rewrite ##e^{a^2+\frac{it}{m\hbar}}## in a different way?
 
yup

e^{(x+b)} =e^{x}.e^{b}

so i am going to have a e^(real part)e^(imaginary part)

absolute value of e^(imaginary) = 1 right (euler's formula)

so the solution is |e^{(real part)}|^2 = e^{(2(a^{2}))}
 
Last edited:
arierreF said:
yup

e^{(x+b)} =e^{x}.e^{b}

so i am going to have a e^(real part)e^(imaginary part)

absolute value of e^(imaginary) = 1 right (euler's formula)

so the solution is |e^{(real part)}|^2 = e^{(2(a^{2}))}

Yes, correct.
 
ok thanks for help
 
Question: A clock's minute hand has length 4 and its hour hand has length 3. What is the distance between the tips at the moment when it is increasing most rapidly?(Putnam Exam Question) Answer: Making assumption that both the hands moves at constant angular velocities, the answer is ## \sqrt{7} .## But don't you think this assumption is somewhat doubtful and wrong?

Similar threads

  • · Replies 7 ·
Replies
7
Views
2K
Replies
9
Views
2K
  • · Replies 1 ·
Replies
1
Views
1K
  • · Replies 2 ·
Replies
2
Views
1K
  • · Replies 1 ·
Replies
1
Views
1K
  • · Replies 5 ·
Replies
5
Views
1K
Replies
1
Views
1K
  • · Replies 16 ·
Replies
16
Views
2K
  • · Replies 5 ·
Replies
5
Views
2K